a and b share the cost in a ratio of 3:2 a pays £125 how much would b pay

Answers

Answer 1

Answer:

[tex]{ \bf{total \: ratio = 3 + 2 = 5}} \\ \\ = { \tt{ \frac{2}{5} \times 125}} \\ = £50[/tex]


Related Questions

Can someone help me?It's urgent and thank you!

Answers

Answer:

y = square root x2 - 5

Step-by-step explanation:

y=[tex]\sqrt{x} -5[/tex]

(the top option)

P(x,y) is a point on the Cartesian plane such that 4x+7<-3 and 5-3y_>11. In which quadrant does the point P lie? Show your working​

Answers

Answer:

3rd quadrant.

Step-by-step explanation:

We know that

4x + 7 < -3

5 - 3*y ≥ 11

We should isolate both of the variables in these inequalities:

for the first one we get:

4x + 7 < - 3

4x < -3 - 7

4x < -10

x < -10/4

So x is smaller than a negative number, then we know that x is negative, from this we already know that the point will be on quadrants 2 or 3.

For the other inequality we get:

5 - 3*y ≥ 11

5 - 11 ≥ 3y

-6 ≥ 3y

-6/3 ≥ y

-2 ≥  y

So we can see that y is equal to or smaller than a negative number, then y is a negative number.

So both values, x and y, are negatives.

This means that the point P(x, y) is in the 3rd quadrant.

The width of a rectangle is 6 units less than the length. The area of the rectangle is 16 units. What is the length, in units, of the rectangle?

Answers

length=8 units

Step-by-step explanation:

LET,A=16 unitsL=a unitsB=a-6 unitsL*B=AReplacing value of L with 'a'. and B with 'a-6',a(a-6)=16a*a-a*6=16a^2-6a=16a^2-6a-16=0a^2-8a+2a-16=0a(a-8)+2(a-8)=0(a-8)(a+2)=0Either,. a-8=0first value a=8Or,. a+2=0second value a=-2(Rejected due to negative)therefore, a=L=8Length=8 units

Translate this word problem as a system of equations and then solve using substitution.

The sum of two numbers is 84. One number is three times the other. Find the numbers.

Put your answer in this form:
The numbers are 3 and 6.

Answers

Answer:

The numbers are 21 and 63

Step-by-step explanation:

let one number be x

let the other number be 3 times the other = 3x

3x + x = 84

4x = 84

x = 21

3x = 3×21 = 63

If R(x) = 2 – 3x – 1, find R(-2)

a. -3
b. 9
c. -9
d. -11

PLEASE HELP

Answers

Answer:

C

Step-by-step explanation:

C

The output of  [tex]R(x) = x^{2} - 3x- 1[/tex] when x = -2 is 9.

What is PEMDAS?

PEMDAS exists as an acronym for the terms parenthesis, exponents, multiplication, division, addition, and subtraction.

To estimate the value of R(-2)

Substitute the value of x = -2, then

[tex]R(-2) = (-2)^{2} - 3(-2) - 1[/tex]

By using the PEMDAS order of operations

Calculate exponents, [tex](-2)^2 = 4[/tex]

= 4 - 3(-2) - 1

Multiply and divide (left to right), 3(-2) = -6

= 4 - (-6) - 1

Add and subtract (left to right),

4 - (-6) - 1 = 9

Therefore, the value of R(-2) = 9.

To learn more about the value of a function refer to:

https://brainly.com/question/15712340

#SPJ2

Help me please I’m gonna cry finals i have 15 min left please I gave u all my points

3 questions

Answers

Answer:

Your just now have finals ?

Step-by-step explanation:

Ay, good luck tho homie !

Kayleigh has $4500 in a savings account at the bank that earns 0.8% interest per year. How much
interest will she earn in 3 years?

Answers

Answer:

Kayleigh will have a total of $4608.

Step-by-step explanation:

First, you use the formula, I=PRT (Interest=Principal, Rate, Time), then you distribute the numbers: (I=4500x0.8%x3) when you multiply them all, you get $108, then you lastly add 108 to 4500, and you get your final answer of $4608.

Kayleigh will earn $108 in interest over a period of 3 years.

Interest is the additional amount of money that is charged or earned on a principal amount of money. It is typically expressed as a percentage of the principal and is either charged when borrowing money or earned when investing or saving money.

To calculate the interest Kayleigh will earn in 3 years, we can use the formula for simple interest:

Interest = Principal x  Rate x Time

Given:

Principal = $4500

Rate = 0.8% = 0.008 (decimal form)

Time = 3 years

Plugging the values into the formula, we have:

Interest = $4500 x 0.008 x 3

Calculating the expression, we find:

Interest = $108

Therefore, Kayleigh will earn $108 in interest over a period of 3 years.

To know more about simple interests follow

https://brainly.com/question/29785550

#SPJ2

Continue the number pattern for the next three terms 1,8,27.....,........,..........

Answers

Answer:

64, 125, 216

Step-by-step explanation:

1, 8, and 27 are perfect cubes, i.e., each is the cube of an integer, and they're perfect cubes of consecutive positive integers:

1 = 1³ = 1×1×1

8 = 2³  

27 = 3³

Therefore, the next three-term numbers of the given sequence are:

64 =  4³

125 = 5³ and

216 = 6³

Therefore, the final sequence will look like this:

1, 8, 27, 64, 125, 216

explain by step by step pls :( if u type something wrong ill report u

Answers

Answer:

∠ C = ∠BCD = 30°

Step-by-step explanation:

∠ EDF = ∠GFC  = 110°                [ corresponding angles ]

Now consider triangle BCF

∠FBC  + ∠ABC = 180°                [ straight line angle ]

∠FBC  + 100° = 180°

∠FBC  = 180 - 100 = 80°

∠BFC + ∠GFC = 180°                  [ straight line angle ]

∠BFC + 110° = 180°

∠BFC = 180 - 110 = 70°

Sum of angles of triangle is 180°

Therefore , in triangle BCF

That is ,

       ∠F + ∠B + ∠ C  = 180°

          70° + 80° + ∠C = 180°

           ∠C = 180 - 150 = 30°            

       

Al lanzar un dado dos veces consecutivas. ¿Qué probabilidad hay de obtener primero un 3 y luego un numero par?

Answers

Answer:

The probability is 1/12.

Step-by-step explanation:

Number of elements in sample space is 6 . Even numbers are 2, 4 and 6 so the there are 3 three even numbers.

So, the probability of getting 3 on the first chance and then an even number in the second chance is

[tex]P = \frac{1}{6}\times \frac{3}{6}\\\\P = \frac{1}{12}[/tex]

the length of a pond is 1700 CM breadth is 14m and height is 1000 CM if a point is half filled calculate the volume of a water in the pond ​

Answers

Answer:

1190 m^3

Step-by-step explanation:

l = 1700 cm = 17 m

b = 14 m

h = 1000 cm = 10 m

Total volume = l × b × h

= 17 × 14× 10

= 2380 m^3

since it is half filled ,

Volume is half , so,

volume of water in pond = 2380 ÷ 2

= 1190 m^3

Ana María tiene $ 30.000 en su cuenta vista y retira $ 18.000 para un pago. Luego el Banco paga un cheque de $ 26.000 con cargo a la cuenta de Ana María y finalmente le descuentan una comisión por mantenimiento de la cuenta equivalente a $ 2.500. ¿Cuál es el saldo de su cuenta?

Answers

Answer:

El saldo de la cuenta de  Ana María es de -$16.500 , es decir, Ana María debe al banco $16.500.

Step-by-step explanation:

Ana María tiene $ 30.000 en su cuenta vista y retira $ 18.000 para un pago. Para saber la cantidad de dinero que posee en su cuenta luego de retirar el dinero debes realizar la diferencia (resta) entre lo que tiene inicialmente y la cantidad que retira:

$30.000 - $18.000= $12.000

Ahora el Banco paga un cheque de $ 26.000 con cargo a la cuenta de Ana María, por lo que debe restarse la mencionada cantidad a la suma de dinero que Ana María posee luego del retiro de dinero [un cheque con cargo a la cuenta de una persona indica que para el titular de la misma significa una salida de fondos, y por tanto, una disminución de su saldo]:

$12.000 - $26.000= - $14.000

Finalmente le descuentan una comisión por mantenimiento de la cuenta equivalente a $ 2.500, por lo que se debe realizar la diferencia entre la cantidad calculada previamente y $2.500:

- $14.000 - $2.500= -$16.500

El saldo de la cuenta de  Ana María es de -$16.500 , es decir, Ana María debe al banco $16.500.

Factorise a² - b²
.......

Answers

(a-b)(a+b)

Regards,
ArmyCee:)

[tex]\longrightarrow{\green{ (a + b)(a - b) }}[/tex] 

[tex]\sf \bf {\boxed {\mathbb {STEP-BY-STEP\:EXPLANATION:}}}[/tex]

[tex] ={a}^{2} - {b}^{2} [/tex]

[tex] = (a + b)(a - b)[/tex]

[tex]\bold{ \green{ \star{ \orange{Mystique35}}}}⋆[/tex]

21.For an oxidation reaction, the temperature T required is warmer than 50°C. Complete the following inequality by dragging the correct symbol to the box.

a. <
b. >
c. ≤
d. ≥

T __ 50°C

WHAT SYMBOL FILLS OUT THE UNDERSCORE ?!

Answers

The correct answer is T > 50. The temperature (T) must be GREATER/WARMER than the 50 degrees, not less than or equal to.

Someone help please!!!

Answers

Answer:

10%

Step-by-step explanation:

if the total hours of sports being played is 50 hours, and the time playing soccer is 5 hours, you just divide 5 by 50 (technically will be .1, but to make it a percent you multiply by 100)

wich is a solution to (x-3)(x+9)=-27?​

Answers

Answer:

=0=−6

Step-by-step explanation:

2 brothers are 1m 60 cm and 1m 44cm tall. What is their ratio of their height​

Answers

Answer: 10:9

Step-by-step explanation:

brother 1-   1m 60cm

brother 2-  1m 44cm

_________________________________________________________

the ratio would be brother 1 : brother 2 => 1m 60cm : 1m 44cm

_________________________________________________________

convert both the values to cm

1m = 100cm

brother 1 would be 100cm + 60cm or 160cm

brother 2 would be 100cm + 44cm or 144cm

_________________________________________________________

[tex]\frac{160}{144} = \frac{10}{9}[/tex] when simplified

the ratio of brother 1 to brother 2 would be 10:9

Can someone tell me if this is the correct choice?

Answers

Answer:

Step-by-step explanation:

It's not the correct choice. If a root is given, that is the same thing as the solution. We go backwards from a solution to a factor, which is what you need to do here.

If x = 2i is the solution, then the factor is

(x - 2i). Likewise, with the other one.

If x = 3i is the solution, then the factor is

(x - 3i). The leading coefficient of 1 just sits outside the first set of parenthesis, and because multiplication is commutative, it doesn't matter which set you put first. Thus, the one you want looks like this:

f(x) = (x - 2i)(x - 3i) or

f(x) = (x - 3i)(x - 2i).

Condense the expression into a
single logarithm and simplify.
log26 - log24

Answers

Step-by-step explanation:

this is solved like this

[tex] log_{3}(6) \: - log_{3}(4) \\ = log_{3}( \frac{6}{4} ) \\ = log_{3}( \frac{3}{2} ) \\ = log_{3}(1.5) \\ [/tex]

11. Kevin is comparing job offers at two stores.
Dollar Deal offers $8/h plus 10% commission.
Great Discounts offers $18/h with no
commission. What do Kevin's weekly sales
need to be in order for Dollar Deal to pay
more? Assume that he works an 8-h day five
days per week.

Answers

Answer:

Kevin's weekly sales will have to be worth more than $4000 in order for Dollar Deal to pay more.

Step-by-step explanation:

Dollar Deal offers $8/h plus 10% commission.

If he spends x hours, then offer after x hours and s amount from sales is;

8x + 10%s

This can be rewritten as; 8x + 0.1s

Now, Great Discounts offers $18/h with no commission.

Thus after 8 hours for 5 days, total number of hours = 40 hours.

Thus, great discounts offer = 18 × 40 = $720

Now, dollar deal will offer; 8(40) + 0.1s = 320 + 0.1s

Thus, for dollar deal to pay more weekly, then;

320 + 0.1s > 720

0.1s > 720 - 320

0.1s > 400

s > 400/0.1

s > 4000

A steamer travels 36 km upstream and 32 km downstream in 6.5 hours. The same steamer travels 4 km upstream and 40 km downstream in 180 minutes. Determine the steamer's speed in still water and the stream's speed.

Answers

Answer:

The streamer's speed in still water is 90.23 km/h while the stream's speed is 33.33 km/h

Step-by-step explanation:

Let v = streamer's speed in still water and v' = stream's speed. His speed upstream is V = v + v' and his speed downstream is V' = v - v'.

Since he travels 36 km upstream, the time taken is t = 36/V = 36/(v + v').

he travels 32 km downstream, the time taken is t' = 32/V' = 32/(v - v')

The total time is thus t + t' = 36/(v + v') + 32/(v - v')

Since the whole trip takes 6,5 hours,

36/(v + v') + 32/(v - v') = 6.5  (1)

Multiplying each term by (v + v')(v - v'), we have

(v + v')(v - v')36/(v + v') + (v + v')(v - v')32/(v - v') = 6.5(v + v')(v - v')  (1)

(v - v')36 + (v + v')32 = 6.5(v + v')(v - v')  (1)

36v - 36v' + 32v + 32v' = 6.5(v² + v'²)

68v - 4v' = 6.5(v² + v'²)        (2)

Also he travels 4 km upstream, the time taken is t" = 4/V = 4/(v + v').

he travels 40 km downstream, the time taken is t'" = 40/V' = 40/(v - v')

The total time is thus t" + t'" = 4/(v + v') + 40/(v - v')

Since the whole trip takes 180 minutes = 3 hours,

4/(v + v') + 40/(v - v') = 3  (3)

Multiplying each term by (v + v')(v - v'), we have

(v + v')(v - v')4/(v + v') + (v + v')(v - v')40/(v - v') = 3(v + v')(v - v')  (1)

(v - v')4 + (v + v')40 = 3(v + v')(v - v')  (1)

4v - 4v' + 40v + 40v' = 3(v² + v'²)

44v - 36v' = 3(v² + v'²)      (4)

Dividing (2) by (4), we have

(68v - 4v')/(44v - 36v') = 6.5(v² + v'²)/3(v² + v'²)      

(68v - 4v')/(44v - 36v') = 6.5/3

3(68v - 4v') = 6.5(44v - 36v')

204v - 12v' = 286v - 234v'

204v - 286v = 12v' - 234v'

-82v = -222v'

v = -222v'/82

v = 111v'/41

Substituting v into (2), we have

68v - 4v' = 6.5(v² + v'²)      

68(111v'/41) - 4v' = 6.5[(111v'/41)² + v'²]        

[68(111/41) - 4]v' = 6.5[(111/41)² + 1]v'²    

[68(111/41) - 4]v' = 6.5[(111/41)² + 1]v'²

[7548/41 - 4]v' = 6.5[12321/1681 + 1]v'²

[(7548 - 164)/41]v' = 6.5[(12321 + 1681)/1681]v'²

[7384/41]v' = 6.5[14002/1681]v'²

[7384/41]v' = [91013/1681]v'²

[91013/1681]v'² - [7384/41]v' = 0

([91013/1681]v' - [7384/41])v' = 0

⇒ v' = 0 or ([91013/1681]v' - 7384/41) = 0

⇒ v' = 0 or [91013/1681]v' - 7384/41) = 0

⇒ v' = 0 or v' =  7384/41 × 16841/91013

⇒ v' = 0 or v' =  180.097 × 0.185

⇒ v' = 0 or v' =  33.33 km/h

Since v' ≠ 0, v' = 33.33 km/h

Substituting v' into v = 111v'/41 = 111(33.33 km/h)/41 = 3699.63 km/h ÷ 41 = 90.23 km/h

So, the streamer's speed in still water is 90.23 km/h while the stream's speed is 33.33 km/h

what is equivalent to 64 1/4

Answers

64.25 is equivalent to that factor


What is the measure of EGF?


What is the measure of CGF?




Answers

Answer:

65 degree, 15 degree

Step-by-step explanation:

In triangle GEF,

∠E = ∠G (Given)

Let's say,

∠E = ∠G = x

∠E + ∠G + ∠F = 180 (Angle sum property of Triangle)

=>x + x + ∠F = 180

=>2x + ∠F = 180

=>2x + 50 = 180

=>2x = 180-50

=> 2x =130

=> x = 130/2

=> x = 65

=>∠E = ∠G = 65

=>∠GEF = ∠EGF = 65 Degree

∠EGF + ∠CGF = 180

=>65 + ∠CGF =180

=>∠CGF = 180 - 65

=> ∠CGF = 15

A scale model of a sculpture has a scale of 2:53
The height of the model is 20cm.
Find the height of the actual sculpture.
Give your answer in m.

Answers

Answer:

ok so we can simpley this is 1:106

so we multiply

20*106=2120 this is cm so we make it meters...

21.2meters!!!

Hope This Helps!!!

write an equation for the sentence: the sum of six and twice a number is equal to sixteen minus the number.
URGENT! ONLY HAVE 5 MINUTES!

Answers

Answer:

6+2x=16-x.

Step-by-step explanation:

X will represent the missing number. Sum means 6, twice that number means multiply it by two, equal represents =, minus represents this, therefor the equation 6+2x=16-x represents the correct answer.

Find the volume of the composite figure. Round to the nearest hundredth. NO LINKS PLEASE.

Answers

I think it's a if it wrong my bad

Answer:

b. 113.04 ft³

Step-by-step explanation:

To find the volume of this composite figure, we must find the volumes of the shapes it is composed of. After finding the volumes, add the volumes.

This figure is composed of a circular cone and a cylinder.

To find the volume of a cylinder, multiply the radius by itself twice, then multiply the product pi, lastly multiply the product by its height.

[tex]v=\pi *r^2*h[/tex]

To find the volume of a circular cone, multiply the radius by itself twice, then multiply the product by the height, then multiply the product by pi, lastly multiply the product by 1/3.

[tex]v=1/3*h*\pi* r^2[/tex]

Now that we know the formulas, we can find the volume of this figure.

----------------------------------------------------------------------

The height of the cylinder is 3 feet. The radius is 3 feet as well.

Remember: 3.14 will be used for pi (π).

[tex]v=\pi *3^2*3=84.78[/tex]

The volume of the cylinder is 84.78 feet.

----------------------------------------------------------------------

Further more, we find the volume of the circular cone.

The height of the cone is 3 feet. The radius is 3 feet as well.

[tex]v=1/3*3*\pi *3^2=28.26[/tex]

Lastly, we add the volumes of both shapes.

[tex](84.78+28.26)[/tex]

[tex]=113.04.[/tex]

113.04 does not need to be rounded to the nearest hundredth.

----------------------------------------------------------------------

Therefore, the volume of this composite figure is 113.04 ft.

A scale model of a sculpture has a scale of 2:49.
The height of the model is 24 cm.
Find the height of the actual sculpture.
Give your answer in m.

Answers

Answer:

ok so if the scale is 2:49 that can be simplified as 1:98

so that means the real one is 98 times bigger then 24

98*24= 2352 cm or 196 feet

Jada asked some students at her school how many hours they spent watching television last week, to the nearest hour. Here are a box plot and a histogram for the data she collected.

If anyone trolls imma be very sad :(

Answers

Answer:

100

Step-by-step explanation:

Based on the histogram given :

The vertical axis gives the number of student in the school :

For the range :

0 - 5 = 40

5 - 10 = 30

10 - 15 = 20

15 - 20 = 5

20 - 25 = 3

25 - 30 = 2

Taking the sum :

(40 + 30 + 20 + 5 + 3 + 2) = 100

2+2-2-5+5+6+4+8-2+47+28​

Answers

Answer:

93

Step-by-step explanation:

So, you basically basic math it, or calculator.

Find the length of the missing

Answers

Answer:

12

Step-by-step explanation:

using pythagoras theorem

here 15 is hypotenuse since it is opposite 0f 90 degree

9 and x are the other smaller sides of a triangle

according to pythagoras thorem the sum of square of two smaller sides of a triangle is equal to the square of hypotenuse. So,

9^2 + x^2 = 15^2

81 + x^2 = 225

x^2 = 225 - 81

x^2 = 144

x = [tex]\sqrt{144[/tex]

x = 12

Other Questions
Which equation represents the graphed function? Dave collects model trains. So far he has 23 distinct train cars. How many ways can he put 7 cars together? = Answer 23 PAnswer 7 =23!(237)! =23!16! = Answer 23 Answer 22 21201918 Answer 17 = Answer Assume that Commonwealth Edison Company deposited $250 million in an escrow account with Northern Trust Company at the beginning of 2017 as a commitment toward a power plant to be completed December 31, 2020. How much will the company have on deposit at the end of 4 years if interest is 10%, compounded semiannually How many dust storms occured in 1932? helpppppppWhich equation represents the slope-intercept form of the line below? 5 y-intercept = (0,2) slope = HOME O A. y=2x+ 3 o B. y=-x-2 3 O C. y = 2x - 3 O D. y=-3x+2 kkklkkklkklklkklklklkklkl Please help Ill give brainliest Find the equation of the straight line that passes through the points (1, 10) and (3, 2)ANSWER ASAP Classify each as true or false. if the statement is true, identify the theorem. if the statement is false, give a counter example to show it is false. Which is the main function of carbohydrates in the body? to determine the distance between two cities on a map, which map feature would a student have to use? Compute the value of price elasticity of supply. If the percentage in quantity supply is 75% and percentage change in price is 55%. The young, coiled-up leaf in true ferns is called what The total population of ethnic Southeast Asian students from Burma, Laos, Thailand, Vietnam, and Kampuchea at your university is currently 100 students. The administration of the university wants to increase the diversity of the student body and decides to increase the number of students from Southeast Asia by 8 percent each year, (over the prior year's enrollment) for a period of 15 years. How many students from Southeast Asia will be enrolled at your university after the 15 year period ? The pressure difference, , across a partial blockage in an artery (called a stenosis) is approximated by the equation where is the blood velocity, the blood viscosity , the blood density , the artery diameter, the area of the unobstructed artery, and the area of the stenosis. Determine the dimensions of the constants and . Would this equation be valid in any system of units Someone please help me i really need it!!!! que es una licuadora f(x) = x2. What is g(x)?5[ f(x)g(x)(3,3)X-55x2O A. g(x)=1-02-GB.XO c. g(x) = 3O D. g(x)=-* x2Please help :/ A farmer contracted to sell 100,000 bushels of wheat to a buyer. When the wheat arrived at the destination, the buyer discovered that the farmer had delivered only 96,000 bushels. The buyer sued the farmer for breach of contract. At the trial of the case, the court found that the written contract was intended as a complete and exclusive statement of the terms of the agreement. The farmer offered to prove that in the wheat business, a promise to deliver a specified quantity is considered to be satisfied if the delivered quantity is within 5% of the specified quantity. The buyer objected to the offered evidence. Is the court likely to admit the evidence offered by the farmer?Answers:A. No, because the offered evidence is inconsistent with the express language of the agreement.B. No, because the written contract was totally integrated.C. Yes, because the offered evidence demonstrates that the farmer substantially performed the contract.D. Yes, because the offered evidence explains or supplements the agreement by usage of trade.